LSAT and Law School Admissions Forum

Get expert LSAT preparation and law school admissions advice from PowerScore Test Preparation.

User avatar
 Dave Killoran
PowerScore Staff
  • PowerScore Staff
  • Posts: 5850
  • Joined: Mar 25, 2011
|
#43477
Complete Question Explanation
(The complete setup for this game can be found here: lsat/viewtopic.php?t=16424)

The correct answer choice is (B)

According to the Not Laws, K and O cannot be assigned to supervise tennis. Accordingly, answer choices (A) and (E) can be eliminated. Since H is assigned to swimming, H cannot be assigned to tennis, and answer choice (C) can be eliminated. Answer choice (D) is incorrect since J and N cannot supervise the same activity. Thus, answer choice (B) is proven correct by process of elimination.

Get the most out of your LSAT Prep Plus subscription.

Analyze and track your performance with our Testing and Analytics Package.